LSAT and Law School Admissions Forum

Get expert LSAT preparation and law school admissions advice from PowerScore Test Preparation.

 Administrator
PowerScore Staff
  • PowerScore Staff
  • Posts: 8916
  • Joined: Feb 02, 2011
|
#26238
Complete Question Explanation
(The complete setup for this game can be found here: lsat/viewtopic.php?t=10888)

The correct answer choice is (B)

To answer this Global, Could Be True—EXCEPT question, we need to identify an answer choice that cannot be true. The wording of each answer choice suggests, once again, that the question is designed to test your understanding of the numerical limitations applicable to each bonus group:
June15_game_1_#6_diagram_1.png
Answer choice (A) is incorrect, because the $1k and $3k groups could each have two employees assigned to them:
June15_game_1_#6_diagram_2.png
Answer choice (B) is the correct answer choice. If at most two employees can receive $1k bonuses (K and P) and at least two must receive $3k bonuses (V and Z), the number of employees who receive $1k bonuses can never exceed the number of employees who receive $3k bonuses.

Answer choice (C) is incorrect, because as many employees can receive $1k bonuses as can receive $5k bonuses. Here’s a hypothetical example:
June15_game_1_#6_diagram_3.png
You can also use the local diagram for Question #5 to show that answer choice (C) could be true and is therefore incorrect.

Answer choice (D) is incorrect, because it is possible that more employees receive $1k bonuses than receive $5k bonuses:
June15_game_1_#6_diagram_4.png
Answer choice (E) is incorrect, because it is possible that more employees receive $3k bonuses than receive $5k bonuses. This is clearly shown in the local setups for Questions #2 and #4.
You do not have the required permissions to view the files attached to this post.

Get the most out of your LSAT Prep Plus subscription.

Analyze and track your performance with our Testing and Analytics Package.